Menu Close

lim-n-k-1-n-sin-1-n-k-




Question Number 164122 by qaz last updated on 14/Jan/22
lim_(n→∞) Σ_(k=1) ^n sin (1/(n+k))=?
$$\underset{\mathrm{n}\rightarrow\infty} {\mathrm{lim}}\underset{\mathrm{k}=\mathrm{1}} {\overset{\mathrm{n}} {\sum}}\mathrm{sin}\:\frac{\mathrm{1}}{\mathrm{n}+\mathrm{k}}=? \\ $$
Answered by mathmax by abdo last updated on 15/Jan/22
we know  x−(x^3 /6)≤sinx≤x ⇒(1/(n+k))−(1/(6(n+k)^3 ))≤sin((1/(n+k)))≤(1/(n+k))  ⇒Σ_(k=1) ^n  (1/(n+k))−(1/6)Σ_(k=1) ^n  (1/((n+k)^3 ))≤Σ_(k=1) ^n sin((1/(n+k)))≤Σ_(k=1) ^n  (1/(n+k))  but Σ_(k=1) ^n  (1/(n+k))=(1/n)Σ_(k=1) ^n  (1/(1+(k/n)))→∫_0 ^1  (dx/(1+x))=ln2  n+k≥n+1>n ⇒(1/(n+k))<(1/n) ⇒(1/((n+k)^3 ))<(1/n^3 ) ⇒Σ_(k=1) ^n  (1/((n+k)^3 ))<(1/n^2 )→0 ⇒  lim_(n→+∞) Σ_(k=1) ^n  sin((1/(n+k)))=ln(2)
$$\mathrm{we}\:\mathrm{know}\:\:\mathrm{x}−\frac{\mathrm{x}^{\mathrm{3}} }{\mathrm{6}}\leqslant\mathrm{sinx}\leqslant\mathrm{x}\:\Rightarrow\frac{\mathrm{1}}{\mathrm{n}+\mathrm{k}}−\frac{\mathrm{1}}{\mathrm{6}\left(\mathrm{n}+\mathrm{k}\right)^{\mathrm{3}} }\leqslant\mathrm{sin}\left(\frac{\mathrm{1}}{\mathrm{n}+\mathrm{k}}\right)\leqslant\frac{\mathrm{1}}{\mathrm{n}+\mathrm{k}} \\ $$$$\Rightarrow\sum_{\mathrm{k}=\mathrm{1}} ^{\mathrm{n}} \:\frac{\mathrm{1}}{\mathrm{n}+\mathrm{k}}−\frac{\mathrm{1}}{\mathrm{6}}\sum_{\mathrm{k}=\mathrm{1}} ^{\mathrm{n}} \:\frac{\mathrm{1}}{\left(\mathrm{n}+\mathrm{k}\right)^{\mathrm{3}} }\leqslant\sum_{\mathrm{k}=\mathrm{1}} ^{\mathrm{n}} \mathrm{sin}\left(\frac{\mathrm{1}}{\mathrm{n}+\mathrm{k}}\right)\leqslant\sum_{\mathrm{k}=\mathrm{1}} ^{\mathrm{n}} \:\frac{\mathrm{1}}{\mathrm{n}+\mathrm{k}} \\ $$$$\mathrm{but}\:\sum_{\mathrm{k}=\mathrm{1}} ^{\mathrm{n}} \:\frac{\mathrm{1}}{\mathrm{n}+\mathrm{k}}=\frac{\mathrm{1}}{\mathrm{n}}\sum_{\mathrm{k}=\mathrm{1}} ^{\mathrm{n}} \:\frac{\mathrm{1}}{\mathrm{1}+\frac{\mathrm{k}}{\mathrm{n}}}\rightarrow\int_{\mathrm{0}} ^{\mathrm{1}} \:\frac{\mathrm{dx}}{\mathrm{1}+\mathrm{x}}=\mathrm{ln2} \\ $$$$\mathrm{n}+\mathrm{k}\geqslant\mathrm{n}+\mathrm{1}>\mathrm{n}\:\Rightarrow\frac{\mathrm{1}}{\mathrm{n}+\mathrm{k}}<\frac{\mathrm{1}}{\mathrm{n}}\:\Rightarrow\frac{\mathrm{1}}{\left(\mathrm{n}+\mathrm{k}\right)^{\mathrm{3}} }<\frac{\mathrm{1}}{\mathrm{n}^{\mathrm{3}} }\:\Rightarrow\sum_{\mathrm{k}=\mathrm{1}} ^{\mathrm{n}} \:\frac{\mathrm{1}}{\left(\mathrm{n}+\mathrm{k}\right)^{\mathrm{3}} }<\frac{\mathrm{1}}{\mathrm{n}^{\mathrm{2}} }\rightarrow\mathrm{0}\:\Rightarrow \\ $$$$\mathrm{lim}_{\mathrm{n}\rightarrow+\infty} \sum_{\mathrm{k}=\mathrm{1}} ^{\mathrm{n}} \:\mathrm{sin}\left(\frac{\mathrm{1}}{\mathrm{n}+\mathrm{k}}\right)=\mathrm{ln}\left(\mathrm{2}\right) \\ $$

Leave a Reply

Your email address will not be published. Required fields are marked *